Rolle's Theorem: Proving Fixed & Non-Fixed Cases

  • Thread starter Thread starter sergey_le
  • Start date Start date
  • Tags Tags
    Theorem
Click For Summary
The discussion focuses on proving Rolle's Theorem by dividing the function into two cases: when it is fixed and when it is not fixed within a segment. The challenge arises in the non-fixed case, where the function is not guaranteed to be differentiable, suggesting the use of the intermediate value theorem instead. A proposed method involves assuming the function is injective and analyzing the midpoint of the interval to demonstrate contradictions in the function's behavior. The conversation emphasizes the need to restrict the choices of points to avoid overlap, ensuring a clearer argument. Overall, the discussion seeks to formalize the proof while navigating the complexities of the function's properties.
sergey_le
Messages
77
Reaction score
15
Homework Statement
Let function ƒ be Continuous in the compact interval [a,b].
if f(a)=f(b), then f is not Injective in the open interval (a,b)
Relevant Equations
Rolle's theorem
It all makes sense to me, but I don't know how to formalize it nicely.
I wanted to divide it into two cases.
First case where f is fixed in the segment.
And a second case where f is not fixed in the segment.
But I don't know how to prove it for the case where f i is not fixed
 
Physics news on Phys.org
Rolle's theorem won't be useful here because you are not given that ##f## is differentiable. This looks like another application of the intermediate value theorem.

Hint: Assume to the contrary that ##f## is injective on ##]a,b[##. Look at the point ##x= (a+b)/2## (or any other point in ##]a,b[##, but the middle of the interval seems like the canonical choice). You know that ##f(x) \neq f(a)##, so either ##f(x) > f(a)## or ##f(x) < f(a)##. Assume without loss of generality ##f(x) > f(a)##.

The intuition now is that to get from ##f(a)## to ##f(x)## you cross the interval ##[f(a), f(x)]## and to get from ##f(x)## to ##f(b)## you cross the interval ##[f(b), f(x)]## again with other ##x##-values. The intermediate value theorem tells that this is impossible because we assumed that ##f## is injective.

Try to make this argument formal now.
 
Last edited by a moderator:
  • Like
Likes mathwonk
Math_QED said:
Rolle's theorem won't be useful here because you are not given that ##f## is differentiable. This looks like another application of the intermediate value theorem.

Hint: Assume to the contrary that ##f## is injective on ##]a,b[##. Look at the point ##x= (a+b)/2## (or any other point in ##]a,b[##, but the middle of the interval seems like the canonical choice). You know that ##f(x) \neq f(a)##, so either ##f(x) > f(a)## or ##f(x) < f(a)##. Assume without loss of generality ##f(x) > f(a)##.

The intuition now is that to get from ##f(a)## to ##f(x)## you cross the interval ##[f(a), f(x)]## and to get from ##f(x)## to ##f(b)## you cross the interval ##[f(b), f(x)]## again with other ##x##-values. The intermediate value theorem tells that this is impossible because we assumed that ##f## is injective.

Try to make this argument formal now.
Ok, I sort of understand what you want me to do.
You want me to divide the interval [a,b] Into two parts With the help of a midpoint c.
And say that For any t that is between f(a) and f(c) Exists x1∈[a,c] so that f(x1)=t , Then switch the point f(a) whit f(b) And say that ∃x2∈[c,b] so that f(x2)=t .
But there can be a situation where x1=c=x2 and than f(x1) and f(x2) That's the same point.
I hope you understand me
 
sergey_le said:
Ok, I sort of understand what you want me to do.
You want me to divide the interval [a,b] Into two parts With the help of a midpoint c.
And say that For any t that is between f(a) and f(c) Exists x1∈[a,c] so that f(x1)=t , Then switch the point f(a) whit f(b) And say that ∃x2∈[c,b] so that f(x2)=t .
But there can be a situation where x1=c=x2 and than f(x1) and f(x2) That's the same point.
I hope you understand me

Restrict your ##x_1## choices. Choose them in ##]a,c[## and not in ##[a,c]##. Similarly for the other interval.
 
  • Like
Likes sergey_le
Math_QED said:
Restrict your ##x_1## choices. Choose them in ##]a,c[## and not in ##[a,c]##. Similarly for the other interval.
Thanks
 
Question: A clock's minute hand has length 4 and its hour hand has length 3. What is the distance between the tips at the moment when it is increasing most rapidly?(Putnam Exam Question) Answer: Making assumption that both the hands moves at constant angular velocities, the answer is ## \sqrt{7} .## But don't you think this assumption is somewhat doubtful and wrong?

Similar threads

  • · Replies 1 ·
Replies
1
Views
2K
  • · Replies 2 ·
Replies
2
Views
2K
  • · Replies 12 ·
Replies
12
Views
2K
  • · Replies 6 ·
Replies
6
Views
1K
  • · Replies 2 ·
Replies
2
Views
2K
  • · Replies 2 ·
Replies
2
Views
1K
Replies
2
Views
1K
  • · Replies 9 ·
Replies
9
Views
3K
  • · Replies 2 ·
Replies
2
Views
2K
  • · Replies 5 ·
Replies
5
Views
1K